LSAT and Law School Admissions Forum

Get expert LSAT preparation and law school admissions advice from PowerScore Test Preparation.

 Administrator
PowerScore Staff
  • PowerScore Staff
  • Posts: 8917
  • Joined: Feb 02, 2011
|
#24912
Complete Question Explanation

Assumption. The correct answer choice is (A)

While this argument is fairly straightforward, its order of presentation in the stimulus is a bit
confusing. The conclusion is in the middle of the stimulus, and is not introduced by a traditional
indicator word. Although, notice that it is introduced by the phrase “this is why...,” which establishes
the first sentence as a premise that supports the first clause of the second sentence. To remove this
structural point of confusion, we can reorder the argument, which provides a rule, a fact, and then a
conclusion that results from the application of the rule to the fact:
  • Rule: ..... the profitability of a business is reduced by anything that undermines
    employee morale

    Fact: ..... paying senior staff with stock options, which allows them to earn more when
    the enterprise prospers, increases dramatically the difference in income
    between senior staff and employees, who are paid only a fixed salary

    Conclusion: ..... thus, paying senior staff with stock options is not a wise policy
The stimulus makes clear that the conclusion results from an application of the rule to the fact, but
there is a problem. The argument makes no explicit connection between employee morale and a
dramatic difference in pay between senior staff and fixed-salary employees.
The question stem identifies this as an Assumption question. Given the logical gap described
above, we can prephrase that the correct answer choice will connect the rule to the fact, by telling
us that having a such a dramatic difference in pay between senior staff and fixed-salary employees
undermines employee morale. This is a causal relationship:

DD = dramatic difference in income between senior staff and fixed-salary employees
REM = reduced employee morale
  • Cause ..... ..... Effect
    DD ..... :arrow: ..... REM
Making this connection is necessary for the conclusion that paying senior staff with stock options,
which ultimately will reduce employee morale and the profitability of the business, is not a wise move.

Answer choice (A): This is the correct answer choice, because it explicitly provides the causal
link the author presumed to be present connecting large income differences between senior staff and
fixed-salary employees and the undermining of employee morale.

Answer choice (B): The stimulus tells us that anything that reduces morale reduces profitability.
Whether it is low employee morale or some other cause that usually reduces the profitability of a
business is irrelevant.

Answer choice (C): This answer choice is tricky, because it actually supports the conclusion. The
fact that businesses that pay senior staff with stock options are less profitable than other firms
supports the conclusion that it is unwise to pay senior staff with stock options. However, the correct
answer choice to an Assumption question does not support the conclusion—it is required for the
conclusion to be valid. In this case, the author made clear that the rule in the first sentence was the
reason why paying senior staff with stock options is unwise. The comparison this answer choice
makes between businesses that pay senior staff in stock options and those that do not is irrelevant to
that connection between morale and the wisdom of paying senior staff in stock options.

Answer choice (D): This answer choice is a reversal of the evidence in the stimulus, that lower
morale decreases profitability, and is not required for the conclusion.

Answer choice (E): This answer choice is tempting because it makes a comparison between
employees who benefit from some form of profit-sharing, as do the senior staff in this question, and
fixed-salary employees. However, the issue was not one of productivity, but morale. The idea of
productivity, while perhaps related to morale, is irrelevant.
 albamolina
  • Posts: 4
  • Joined: Apr 09, 2017
|
#34006
I was down to answers A and D. I selected the correct answer, however, it took me a very long time. I was wondering if there is a trick that could help me attack the question with more ease. How do you spot reversals? What is the rule?
 Kristina Moen
PowerScore Staff
  • PowerScore Staff
  • Posts: 230
  • Joined: Nov 17, 2016
|
#34023
Hi Alba,

Great question. Welcome to the Forum. The conclusion here is that "paying senior staff with stock options, which allows them to earn more when the enterprise prospers, is not a wise policy." Why? Because it increases the difference of income between senior staff and employees and undermining employee morale reduces profitability. Pay attention to those sentences. What's the disconnect? Here, it's "employee morale" and "differences in income."

One trick for an Assumption question is to use the Assumption Negation Technique. If you negate an answer choice, and it kills the conclusion, then it's the winner! You want an answer choice that is required by the argument, not just something that strengthens or even justifies the conclusion (there can be several ways to justify a conclusion, not all of them necessary).

To negate answer choice (A): Large income differences between fixed-salary employees and senior staff do not tend to undermine employee morale. - Well that kills the conclusion. Because it doesn't undermine employee morale, so it wouldn't reduce profitability (at least this way).

To negate answer choice (D): "Reducing the difference in income between senior staff and employees paid only a fixed salary does not invariably increase a company’s profitability." Maybe profitability stays the same. Does that kills the conclusion? No, the argument is about INCREASING the difference in income, not about decreasing it. It's sort of like the difference between LOSING a dollar and GAINING a dollar. Seems the same right? But anyone who has lost their wallet will tell you it's different. So perhaps giving your boss a raise and leaving your income the same will make you less happy, but decreasing your boss's salary won't make you more happy.
 nelson3clan
  • Posts: 19
  • Joined: Apr 23, 2015
|
#38772
Good evening. I actually had contenders of A & B. unfortunately I choose B, which was the wrong answer. I was looking at the assumption being what if it wasn't that profitability was due to low employee morale.
 Adam Tyson
PowerScore Staff
  • PowerScore Staff
  • Posts: 5153
  • Joined: Apr 14, 2011
|
#39082
Hey Nelson, let me see if I can help here. The author is definitely saying that low morale causes low profitability, but is he saying that is the cause most of the time? Not at all! It may be that low profitability is usually (more than half the time) due to a poor quality product or service, or that it is usually caused by poor cost controls, or that it is usually caused by a number of other factors besides or in addition to these. Low morale might be the cause only a small percentage of the time, even if it always causes it.

Try the negation of answer B - "Reductions in the profitability of a company are NOT usually due to low employee morale". What does that do the claim that low morale will always cause low profitability? Not a thing - it still could always cause it, but it still happens relatively infrequently.

Try this analogy - "Getting hit by a bus always makes me go to the hospital." Does that mean that my going to the hospital is usually due to me getting hit by a bus? Nope! Maybe I usually go to visit my sick friend, or because I work there, or for any number of combinations of reasons.

Good luck on your continued studies. Keep your morale up and watch out for buses!
 blade21cn
  • Posts: 100
  • Joined: May 21, 2019
|
#84165
This is the rarest necessary assumption question, where the credited answer choice is a "most" statement - "tend to." Shouldn't it be a "some" statement instead - "at least sometimes undermine ..."? Can anybody elaborate why the "most" language is justified? Thanks!
 Robert Carroll
PowerScore Staff
  • PowerScore Staff
  • Posts: 1787
  • Joined: Dec 06, 2013
|
#84545
blade,

As this is a Necessary Assumption question, the relevant "some" statement would be acceptable, because it's strictly weaker (so if the "most" has to be assumed, the "some" entailed by it also has to be assumed), but "most" is not unacceptably strong here. The author wants to establish that a certain policy is not wise. If it's unwise in "some" situations, the unqualified statement by the author that it's unwise wouldn't make sense. So the author is here committed to its lack of wisdom in more than just some situations.

Incidentally, I looked at a few Necessary Assumption questions, and I found several in the first ten I saw that involved "most" or stronger language, so this does not appear to be a rare case.

Robert Carroll

Get the most out of your LSAT Prep Plus subscription.

Analyze and track your performance with our Testing and Analytics Package.